返回列表 发帖

求助 OG11-7

求助各位og11-7,我怎么看了答案都还没明白什么意思?

7. A researcher discoved that people who have low levels of immune-system activity tend to score much lower on tests of mental health than do people with normal or high immune-system activity, The researcher concluded from this experiment that the immune system protects against mental illness as well as against physical disease.

The researcher's conclusion depends on which of the following assumption?

A)High immune-system activity protects against mental illness better than normal immune-system activity does.

B)Mental illness is similar to physical disease in its effects on body systems.

C) People with high immune-system activity can not develop mental illness.

D)Mental illness does not cause people's immune-system activity to decrease.

E)Psychological treatment of mental illness is not as effective as is medical treatment.

谢谢各位

收藏 分享

就是10th中的48题,这是og的解释:

The researcher concludes from the association of low immune-system activity with low mental-health sores that, in effect, immune system activity can inhibit mental illness. If, contrary to D, mental illness can depress immune-system activity, the association mentioned does not support the researcher’s conclusion. So D must be assumed.

Normal immune-system activity could protect against mental illness without high-immune system activity offering increased protection or prevention, contrary to what A and C state, so neither of A and C is assumed. The conclusion does not depend on there being a similarity between mental and physical illness or a difference in treatments, so B and E are not assumed

原文提及了A (immune-system activity)与B( mental-health sores) 的关系, 然后得出结论就说是A导致B(A---->B)。

那么,要推出这个结论的前提条件就是:B不能导致A( B--->A), 也就是答案D所说的。

两种现象A ,B同时出现,我们不能马上下结论就是前者推后者,因为也有可能是后者推前者。这种错误叫做 因果导致

这种情况有时候也出削弱题。

TOP

谢谢版主!!!!

TOP

返回列表

站长推荐 关闭


美国top10 MBA VIP申请服务

自2003年开始提供 MBA 申请服务以来,保持着90% 以上的成功率,其中Top10 MBA服务成功率更是高达95%


查看